Difference between revisions of "1985 AJHSME Problems/Problem 2"
Megaboy6679 (talk | contribs) m (→Solution 1) |
|||
Line 7: | Line 7: | ||
==Solution 1== | ==Solution 1== | ||
− | + | To simplify the problem, we can group 90’s together: <math>90 + 91 + ... + 98 + 99 = 90 \cdot 10 + 1 + 2 + 3 + ... + 8 + 9</math>. | |
− | |||
− | + | <math>90\cdot10=900</math>, and finding <math>1 + 2 + ... + 8 + 9</math> has a trick to it. | |
− | + | Rearranging the numbers so each pair sums up to 10, we have: | |
− | + | <cmath>(1 + 9)+(2+8)+(3+7)+(4+6)+5</cmath>. <math>4\cdot10+5 = 45</math>, and <math>900+45=\boxed{\text{B} 945}</math>. | |
− | |||
==Solution 2== | ==Solution 2== |
Revision as of 20:28, 1 February 2023
Problem
Solution 1
To simplify the problem, we can group 90’s together: .
, and finding
has a trick to it.
Rearranging the numbers so each pair sums up to 10, we have:
.
, and
.
Solution 2
We can express each of the terms as a difference from 100 and then add the negatives using to get the answer.
Solution 3
Instead of breaking the sum and then rearranging, we can start by rearranging:
Solution 4
We can use the formula for finite arithmetic sequences.
It is (
) where
is the number of terms in the sequence,
is the first term and
is the last term.
Applying it here:
Video Solution
~savannahsolver
See Also
1985 AJHSME (Problems • Answer Key • Resources) | ||
Preceded by Problem 1 |
Followed by Problem 3 | |
1 • 2 • 3 • 4 • 5 • 6 • 7 • 8 • 9 • 10 • 11 • 12 • 13 • 14 • 15 • 16 • 17 • 18 • 19 • 20 • 21 • 22 • 23 • 24 • 25 | ||
All AJHSME/AMC 8 Problems and Solutions |
The problems on this page are copyrighted by the Mathematical Association of America's American Mathematics Competitions.